Exam 2 Practice Questions P2

¡Supera tus tareas y exámenes ahora con Quizwiz!

The nurse is seeing a client at the local community mental health clinic. The client states, "I want to kill myself. I have nothing to live for; no one would miss me." What is the priority question the nurse should ask the client? "Do you have a specific plan for killing yourself?" "When do you plan to kill yourself?" "What has caused you to have such feelings?" "What about your family - don't you care about them?"

"Do you have a specific plan for killing yourself?" Explanation: A person with a specific plan and access to the means is considered to have very "lethal" suicidal ideation.

During the health history inquiry about alcohol intake, which of the following is a CAGE question? "How often do you have a hangover?" "How many days per week do you drink?" "Describe the types of alcohol that you prefer." "Have you ever felt annoyed by criticism about drinking?"

"Have you ever felt annoyed by criticism about drinking?" Explanation: "Have you ever felt annoyed by criticism about drinking?" is one of the 4 questions that make up the CAGE questionnaire.

The nurse is documenting the findings from a mental status examination. After noting the client's appearance and behavior, what should the nurse document next? Loss of short-term memory Demonstration of sadness Ability to articulate verbally Ability to identify today's date

Ability to articulate verbally Explanation: The nurse should follow the mental status examination order when documenting the findings from an examination. After documenting the client's appearance and behavior, the nurse should document content about the client's speech and language. The documentation should then identify the client's mood, thoughts and perceptions, and cognitive function.

Which person would the nurse identify as experiencing polyvictimization? An older adult experiences physical, emotional, and financial abuse from a family member. A 10-year-old girl experiences sexual abuse from family members and friends of the family. A 15-year-old boy experiences punking and bullying in the high school environment. A 27-year-old girl is promised a position as a nanny in a country other than her home country.

An older adult experiences physical, emotional, and financial abuse from a family member. Explanation: Polyvictimization is when an individual experiences more than one type of violence. The older adult is subjected to three types of violence - physical, emotional, and financial abuse. Each of the other individuals is experiencing only one type of violence

The nurse should clarify which medication order prescribed for a client with chronic back pain? Narcotic analgesia Tricyclic antidepressant Selective serotonin reuptake inhibitor Angiotensin converting enzyme inhibitor

Angiotensin converting enzyme inhibitor Explanation: To treat some chronic pain conditions, health care providers may prescribe medications that increase serotonin levels, such as tricyclic antidepressants and selective serotonin reuptake inhibitors, to modulate incoming pain stimuli. Opiates, antidepressants, and calcium channel blockers are pharmacological alternatives in the treatment of chronic pain. ACE inhibitors are not routinely prescribed for chronic pain conditions.

A client is having pain and rates it as 4 on a scale of 0 to 10. The client states that she does not want to take any medication at this time. Using the gate control theory, what nonpharmacological approach could the nurse use to decrease the client's pain? Music therapy Back rub Guided imagery Dim lights

Back rub Explanation: Although all the above are nonpharmacological approaches to pain reduction and work well for some clients, the only one based on blocking pain from entering the gate at the synaptic junction is a back rub. The National Cancer Institute suggests that back rubs may enhance the effects of analgesics and decrease inflammation. Back rubs decrease pain by reducing cortisol levels and increasing serotonin levels, as well as stimulating the release of endorphins.

Which of the following would be best for a nurse to use when assessing for fremitus in a client? Dorsal hand surface Pads of fingers Ball of hand Fist

Ball of hand Explanation: The ball of the hand is best for assessing tactile fremitus because the area is especially sensitive to vibratory sensation. The dorsal surface of the hand is used to assess temperature. The fist is used in blunt percussion. Finger pads are used for fine discrimination such as pulses, texture, and size

The nurse is admitting a client with substance abuse. What screening tool would be best to use if this client is in denial about his substance abuse? CAGE questionnaire Mini-mental status examination Mini-Cog exam SAD PERSONAS

CAGE questionnaire Explanation: When screening for substance abuse, the client may deny the problem. The CAGE tool is valuable because it addresses this denial. The tool is a quick first-step questionnaire to use.

A nurse performs an inspection and palpation of the auricle when examining the ear of a client. Which documentation by the nurse demonstrates a normal finding? Nontender, hard tophi Ulcerated, crusted nodules Pale blue coloration Darwin's tubercle

Darwin's tubercle Explanation: The nurse should document Darwin's tubercle as a normal finding upon examination of the auricle. Darwin's tubercle is a clinically insignificant projection seen on the auricle. Nontender, hard tophi are manifestations of gout. Ulcerated, crusted nodules can be a sign of skin cancer. Pale blue coloration of the auricle could be due to frostbite.

A nurse is auscultating a client's chest for breath sounds. The nurse recognizes that which of the following is the strongest stimulus to breathe? Hypoxemia Hypoventilation Hypercapnia Hyperventilation

Hypercapnia Explanation: Under normal circumstances, the strongest stimulus to breathe is an increase of carbon dioxide in the blood (hypercapnia). A decrease in oxygen (hypoxemia) also increases respiration but is less effective than a rise in carbon dioxide levels. Hypoventilation is a breathing pattern marked by a decreased rate, decreased depth, and irregular pattern. Hyperventilation is a breathing pattern marked by increased rate and depth.

A client rates his pain as "9" on a scale of 1 to 10. The nurse would expect to assess which of the following? Constricted pupils Hypotension Increased serum glucose Flaccid muscles

Increased serum glucose Explanation: Pain elicits a stress response that triggers the sympathetic nervous system resulting in dilated pupils, increased blood pressure, increased serum glucose levels, and muscle spasms.

A nurse is interviewing a client who is a suspected victim of abuse. Which practice should the nurse avoid during this phase of assessment? Using direct questions about being injured Displaying a concerned, empathetic approach Interjecting often to clarify information Emphasizing the nurse's availability to talk

Interjecting often to clarify information Explanation: When interviewing a client, the nurse must remember to ask questions and allow the client to answer completely, without interrupting the client. The questions should be specific, such as; has anyone in your home ever hurt you? The nurse should convey a concerned, nonjudgmental attitude and appropriate empathy.

When assessing the speech of an older adult client, which of the following would the nurse expect to find? Repetitive Rapid Moderately paced Loud tone

Moderately paced Explanation: Normally, in older adults, responses may be slowed but speech should be clear and moderately paced. Slow, repetitive speech is characteristic of depression or Parkinson's disease. Loud, rapid speech may occur in manic phases of bipolar disorder.

Kevin is conducting an interview with a psychiatric-mental health client and notices the client is using made-up words. This is known as which of the following? Neologisms Clangs Word themes Broadcasting

Neologisms Explanation: Clients with thought disorders (e.g., schizophrenia) may rhyme (clang associations) or use made-up words (neologisms).

The nurse is performing a cardinal fields of gaze test on a client who has an inner ear infection. What would be an expected finding? Strabismus Phoria Tropia Nystagmus

Nystagmus Explanation: Nystagmus may be associated with an inner ear disorder. Strabismus or tropia would refer to a constant malalignment of the eyes due to a muscle weakness detected with the corneal light reflex test. Phoria describes a drifting of the eyes due to a mild muscle weakness and is detected only with the cover test.

Where should a nurse place the hands to palpate the submandibular lymph nodes? On the medial border of the mandible At the angle of the mandible on the anterior edge At the posterior base of the skull bone A few centimeters behind the tip of the mandible

On the medial border of the mandible Explanation: The submandibular lymph nodes can be palpated on the medial border of the mandibular bone. Tonsillar nodes are found at the angle of the mandible on the anterior edge of the sternomastoid muscle. The occipital nodes can be palpated at the posterior base of the skull bone. Submental lymph nodes can be palpated a few centimeters behind the tip of the mandible.

The client states, "My husband says I cannot have a job because the woman should stay home and take care of the family. He will be the one to provide for his family financially." What method of power is the husband exerting? Privilege Isolation Intimidation Emotional abuse

Privilege Explanation: Privilege is treating victims like servants; making all big decisions; acting the "man of the castle"; being the one who defines men's and women's roles.

The nurse notes that an older client has white spots on the tympanic membrane of the left ear. What should this finding indicate to the nurse? Acute otitis media Serous otitis media Trauma from a current infection Scars from previous ear infections

Scars from previous ear infections Explanation: White spots on the tympanic membrane are scars from previous ear infections. A red bulging eardrum is an indication of acute otitis media. Yellowish bulging membrane is associated with serous otitis media. A perforated ear drum indicates trauma from a current infection.

The nurse is presenting an educational event for gardeners. When discussing the ears, what would be an important topic to cover? Skin cancer prevention Otalgia Tinnitus Sound control

Skin cancer prevention Explanation: Many melanomas develop near or on the helix of the ear. Teaching clients how to protect themselves from unnecessary sun exposure increases the likelihood of preventative behaviors. Otalgia is an earache. Tinnitus is ringing in the ears. Sound control would be related to environmental loud noises. None of the three would be a topic for gardeners.

The nurse is observing a client's posture and facial expression for evidence of pain. Which of the following would most likely lead the nurse to suspect that the client may be experiencing pain? Attentive listening Slumped posture Eye contact Periodic position changes

Slumped posture Explanation: A slumped posture indicates that the client is disturbed or uncomfortable, suggesting pain. Attentive listening and eye contact are normal findings suggesting no pain. Excessive changes in position would suggest possible pain.

A client is diagnosed with bronchitis. Which of the f'ollowing would the nurse hear on auscultation? Sibilant wheezes Fine crackles Sonorous wheezes Coarse crackles

Sonorous wheezes Explanation: Sonorous wheezes are often heard with bronchitis. Sibilant wheezes are often heard with acute asthma or chronic emphysema. Fine crackles, heard on late inspiration, suggest restrictive disease, such a pneumonia or heart failure. Coarse crackles may indicate pneumonia, pulmonary edema, or pulmonary fibrosis

An adult client tells the nurse that he has been experiencing gradual vision loss. The nurse should ask about the client's diet. determine whether there is a history of glaucoma. check the client's blood pressure. ask the client if he has any known allergies.

check the client's blood pressure. Explanation: Hypertension narrows blood vessels in the retina affecting vision.

During an examination, the client has incomprehensible, illogic speech that changes abruptly from one topic to another. The nurse should refer this client for further evaluation of schizophrenia mania Korsakoff syndrome disorientation

schizophrenia

A nurse is reviewing a depression questionnaire completed by a client. Which of the following would the nurse interpret as being suggestive of depression? "Occasionally I feel like my attention wanders." "I haven't noticed any change in my appetite." "It usually takes me over an hour to fall asleep." "I might wake up once during the night but not often."

"It usually takes me over an hour to fall asleep." Explanation: Based on the depression questionnaire, routinely taking a long time to fall asleep, such as an hour, would suggest possible depression. Difficulty concentrating such that it interferes with even minor decision making, not occasional mind wandering, would be an indicator of depression. Appetite changes, either significantly increased or decreased, would be a signal for possible depression. Sleeping too much, waking up too early, or awaking during the night if frequent or excessive can suggest depression.

Upon examination, the Advanced Practice Nurse finds that a client has otitis media with effusion. What assessment finding is most clearly indicative of this diagnosis? A diffuse cone of light A gray tympanic membrane A perforated tympanic membrane Inflammation in the ear canal

A diffuse cone of light Explanation: A diffuse cone of light indicates otitis media with effusion.

The nurse notes a cyst on the ear of an older adult. Which assessment data is consistent with a cyst? A sac with a membranous lining filled with fluid A hard nodule composed of uric acid crystals Swelling of the external ear canal Redness and bulging of the eardrum

A sac with a membranous lining filled with fluid Explanation: A cyst on the ear would present as a fluid-filled sac. A tophus is a hard nodule composed of uric acid crystals. Redness and bulging of the eardrum is characteristic of otitis media with effusion. Swelling of the external ear canal with inflammation or infection would be referred to as an edematous ear.

Otoscopic examination of a 69-year-old client's tympanic membrane reveals that it is red, bulging, and distorted. The nurse also notes a diminished light reflex. To what should the nurse most likely attribute this assessment finding? Repeated ear infections Trauma Age-related changes Acute otitis media

Acute otitis media Explanation: A red, bulging eardrum coupled with distorted, diminished, or absent light reflex is associated with acute otitis media. Repeated ear infections usually cause the formation of white scar tissue. Trauma causes the accumulation of blood behind the eardrum, which appears blue or dark red.

The nurse is observing an older adult client dress and groom as part of a mental health assessment. Which finding indicates that the client may need further follow-up related to a cerebral vascular accident (CVA)? extreme unilateral neglect unusually meticulous grooming bizarre dress extremely loose clothing held up by a belt

Correct response: extreme unilateral neglect Explanation: The nurse should anticipate further follow-up related to a possible cerebral vascular accident (CVA) if the client exhibits extreme unilateral neglect, as this is characteristic of a CVA. Unusually meticulous grooming may be seen in obsessive-compulsive disorder. Bizarre dress may be seen in schizophrenia or manic disorders. Loose clothing held up by a belt suggests recent weight loss.

A nurse is teaching nursing students about the difference between a general routine screening and a focused specialty assessment. The nurse determines understanding when the student nurses identify which of the following as part of a focused specialty assessment? Select all that apply. Determine the type of headache a client is experiencing. Inspect the size, shape, and configuration of the head. Palpate the trachea to determine position. Assess signs of trigeminal neuralgia. Assess thyroid dysfunction.

Determine the type of headache a client is experiencing. Assess signs of trigeminal neuralgia. Assess thyroid dysfunction. A focused assessment is performed when a problem is observed such as determining the type of headache the client is experiencing, assessing for signs of trigeminal neuralgia, or determining if symptoms demonstrated by client indicate thyroid dysfunction. Inspecting the size, shape, and configuration of the head and palpating the trachea to determine correct position are examples of general routine screening.

The nurse in the emergency department is caring for a 4 year-old brought in by his parents with complaints that the child will not stop crying and pulling at his ear. Based upon information collected by the nurse, which of the following statements applies to a diagnosis of external otitis? External otitis is characterized by pain when the pinna of the ear is pulled. External otitis is usually accompanied by a high fever in children. External otitis is usually related to an upper respiratory infection. External otitis can be prevented by using cotton-tipped applicators to clean the ear.

External otitis is characterized by pain when the pinna of the ear is pulled. Explanation: External otitis is an infection of the external ear. Pain can be elicited when the pinna of the ear is pulled. Fever and accompanying upper respiratory infection occur more commonly in conjunction with otitis media (infection of the middle ear). Cotton-tipped applicators can actually cause external otitis so their use should be avoided.

A nurse has interviewed a client with a mental health disorder who does not speak English. The nurse enlists assistance from an interpreter. What is important for the nurse and interpreter to do after concluding the interview? Have the interpreter stay with the client while the nurse completes her documentation. Encourage the interpreter to discuss the interaction with other members of the client's cultural community. Go into a private conference area and question the interpreter about the communication style and context of the client. Have the interpreter wait in the cafeteria while the nurse notifies the physician about the results of the interview.

Go into a private conference area and question the interpreter about the communication style and context of the client. Explanation: After the interview, the nurse and interpreter should walk out of sight of the client and, in a private area, discuss the communication style and context of the interview. The nurse should establish if the client made sense, if sentences were structured properly and completely, if the client had difficulty with self-expression, if the client was oriented to reality, and if the nurse should be aware of any cultural practices or beliefs.

The nurse is documenting an objective assessment of the client's ears. Which of the following would be the most appropriate documentation? Hearing intact bilaterally on whisper test Hearing intact on right and left with Rinne test No decrease in hearing evident on Weber test Client states experiencing no decrease in hearing

Hearing intact bilaterally on whisper test Explanation: Documentation of the whisper test should be "hearing intact bilaterally on whisper test." Documentation of the Rinne and Weber test results usually validates normal findings (e.g., "No unexpected findings on Weber and Rinne tests."). The report from the client is a subjective assessment finding.

A nurse is preparing a community education session on hearing loss. Which information should the nurse include? Hearing loss can contribute to emotional isolation. All ethnic groups experience hearing loss in the same way. In order to hold a driver's license, hearing must be tested regularly. Hearing loss typically begins after the age of 40.

Hearing loss can contribute to emotional isolation. Explanation: Hearing is a critical sense with which one can experience the world. Loss of hearing is associated with social and emotional isolation. All ethnic groups do not experience hearing loss in the same way. Clients of African descent experience less hearing loss due to higher amounts of melanin in the cochlea. Unlike vision prerequisites for driving, there is no mandate for widespread testing of hearing. Hearing loss can begin at any age, not just after 40.

Johnny is a 5-year-old boy who comes to the clinic for his scheduled injections. Which physical indications would lead you to suspect child abuse? Bruising on the upper portion of his buttocks. Regular pattern of circular, coin-sized reddened areas on his back. Irregular pattern of small, circular reddened areas with healing blisters on various parts of his body. Crying when he sees the needle for the injection.

Irregular pattern of small, circular reddened areas with healing blisters on various parts of his body. Explanation: Abnormal findings consistent with abuse include scars, bruises, burns, welts or swelling on face, breasts, arms, chest, abdomen, or genitalia, including evidence of cigarette or cigar burns.

While inspecting the client's tympanic membrane, the nurse notes a pearly gray and shiny appearance. The nurse would interpret this finding as what? Scarring from previous infections Otitis media Normal tympanic membrane Otitis externa

Normal tympanic membrane Explanation: The tympanic membrane is normally a pearly gray color with a shiny appearance. White spots would indicate scarring. A yellowish bulging membrane would suggest serous otitis media; a red bulging membrane would suggest acute otitis media. Otitis externa does not directly affect the appearance of the tympanic membrane.

In obtaining a history, you note that a client uses the word "largely" repeatedly, to the point of being a distraction to your task. Which word best describes this speech pattern? Clanging Echolalia Confabulation Perseveration

Perseveration Explanation: Perseveration is the repetition of words or ideas. Echolalia differs in that the client repeats what is said to him. Clanging is the repetition of the same sounds in different words. Confabulation is making up a story in response to a question, as sometimes seen in chronic alcohol use with Korsakoff's syndrome.

An adult farm worker presents at the ED after falling out of a hay loft. The client states falling approximately 4 hours ago and did not lose consciousness. The client is experiencing only a mild headache. When asked why the client came to the ED, the client states that he had a clear discharge from the right ear ever since the accident. What should this finding indicate to the nurse? Possible basilar skull fracture Possible otitis externa Possible otitis media Possible mass in the ear

Possible basilar skull fracture Explanation: Hemotympanum, otorrhea, or tympanic membrane rupture may indicate barotrauma from pressure changes or a basilar skull fracture. Otitis externa is a outer ear infection. Otitis media is an inner ear infection. The drainage from the ear is a classic sign of a possible basilar skull fracture. Possible mass in the ear is not consistent with this client's symptoms.

The nurse assesses an older adult using the Mini-Mental Status Exam. The total score obtained is 24. Which interpretation by the nurse is correct? The client is cognitively intact. The client is not depressed. The client is seriously depressed. The client is cognitively impaired.

The client is cognitively intact. Explanation: A score of 24 on the Mini-Mental Status Exam is indicative of cognitive intactness. A score of 23 or lower is indicative of cognitive impairment. The Mini-Mental Status Exam does not assess depression.

The nurse observes the spouse of a client pinch the client's arm when someone talks with the client in the waiting room of the community clinic. What should this observation indicate to the nurse? The spouse is jealous The spouse was trying to get the client's attention The client is not permitted to have contact with others The client was sharing too much information with the other person

The client is not permitted to have contact with others Explanation: Abusive partners often control the activities of the partner and do not allow outside friendships or significant contact with others. There is not information about the situation to determine if the spouse was jealous. Pinching is not required to get someone's attention. Since the exchange was limited in the waiting room it is unlikely that the client was sharing too much information with the other person.

The nurse's assessment of an 81-year-old client's hearing has corroborated her recent history of hearing loss. The nurse questions the client about her use of hearing aids, to which the client responds, "I've got enough frustration in my life without having to fiddle with those." The nurse should suspect which of the following? The client may misunderstand the factors underlying her hearing loss. The client may have had a negative experience with hearing aids in the past. The client may be unable to afford the cost of hearing aids. The client may be unwilling to adhere to treatment regimens.

The client may have had a negative experience with hearing aids in the past. Explanation: The older client may have had a bad experience with certain hearing aids and may refuse to wear one. The client may also associate a negative self-image with a hearing aid. Financial concerns are a realistic possibility, but these are not strongly suggested by the client's statement. Similarly, a misunderstanding of hearing loss may exist, but this is unlikely to underlie the client's forceful statement.

The nurse begins the physical examination of a newly admitted client by assessing the client's mental status. What is the nurse's best rationale for performing the mental status exam early in the assessment? The client will be less anxious, providing the nurse with more accurate and reliable data. The exam can provide clues about the validity of the client's responses now and throughout. The exam provides data about mental health problems that the client may be afraid to report. The client's fears about having a serious illness may be alleviated by the results of the exam.

The exam can provide clues about the validity of the client's responses now and throughout. Explanation: Assessing mental status at the very beginning of the head-to-toe examination provides clues regarding the validity of the subjective information provided by the client during the history and throughout the exam. Thus it is best to determine validity of client responses before completing the entire physical exam only to learn that the client's answers to questions may have been inaccurate. Assessing mental status first will not necessarily lessen a client's anxiety or fears about a serious illness. The exam can provide data about mental health problems. However, this is not the primary reason for performing the exam at the very beginning.

Which action by the nurse is consistent with Weber's test? The nurse activates the tuning fork and places it on the midline of the parietal bone in line with both ears. The nurse strikes the tuning fork and places it on the client's mastoid process to measure bone conduction. The nurse uses a bulb insufflator attached to an otoscope to observe movement of the tympanic membrane. The nurse shields their mouth and whispers a simple sentence approximately 18 inches from the client's ear.

The nurse activates the tuning fork and places it on the midline of the parietal bone in line with both ears. Explanation: Using Weber's test, the nurse activates the tuning fork and places it on the midline of the parietal bone in line with both ears to differentiate the cause of unilateral hearing loss. In Rinne's test, the nurse strikes the tuning fork and places it on the client's mastoid process to measure bone conduction. When examining the inner ear, the nurse uses a bulb insufflator attached to an otoscope to observe movement of the tympanic membrane. In the Whisper test, the nurse shields their mouth and whispers a simple sentence approximately 18 inches from the client's ear.

A client has suspected hearing loss. Which hearing test would be best for the nurse to perform for this client? whisper hearing test rubbing fingers test formal hearing test tuning fork test

formal hearing test Explanation: A formal hearing test is the gold standard for suspected hearing loss. A formal hearing testing is preferred to the whisper hearing test, the rubbing fingers test, and the tuning fork test.

During a physical examination of the head and neck, a client reports frequently feeling cold. What additional questions should the nurse ask for more information about the client's symptoms? (Select all that apply.) "Do you dress more warmly than other people? "Do you use more blankets than others at home? "Do you perspire more than others?" "Do you perspire less than others?" "Have you lost weight recently?"

"Do you dress more warmly than other people? "Do you use more blankets than others at home? "Do you perspire less than others?" Because the client complains of feeling cold, the nurse should focus additional questions to assess for hypothyroidism. These questions would include "Do you dress more warmly than other people?", "Do you use more blankets than others at home?", and "Do you perspire less than other?" The questions "Do you perspire more than others?" and "Have you lost weight recently?" would be appropriate to assess for hyperthyroidism.

The nurse is evaluating the risk for violence of a prenatal client. Which question by the nurse would be most appropriate? "Have you been physically hurt by someone in the past year?" "What do you do to make him want to hurt you?" "Why are you still living with him and letting him hurt you?" "Are you trying to work things out since you are pregnant."

"Have you been physically hurt by someone in the past year?" Explanation: The most appropriate question for the nurse to ask is, "Have you been physically hurt by someone in the past year?" The other options are judgmental and to facilitate therapeutic communication between the nurse and client.

An adult client tells the nurse that his 80-year-old father is almost completely deaf. After an explanation to the client about risk factors for hearing loss, the nurse determines that the client needs further instruction when the client says "There is a genetic predisposition to hearing loss." "Certain cultural groups have a higher rate of hearing loss." "It is difficult to prevent hearing loss or worsening of hearing." "Chronic otitis media has been associated with hearing loss."

"It is difficult to prevent hearing loss or worsening of hearing." Explanation: Causes of hearing disorders are many and include genetics, infections, injuries to head or ear, ototoxic drugs, aging, and loud noises.

A nurse is conducting a focused ear and hearing assessment of an adult client who has a history of mild hearing loss. When performing the whisper test, what instruction should the nurse begin with? "Please close your eyes and listen carefully to what I say." "Please cover your ear that has the weakest hearing." "Please tell me when you can hear me talking." "Please repeat the words that I say."

"Please cover your ear that has the weakest hearing." Explanation: The whisper test begins by asking the client to occlude the ear in which they have poorer hearing. This precedes the actual test of hearing ability.

The nurse is preparing to assess a client's remote memory. Which question would be most appropriate for the nurse to use? "Can you tell me what you have eaten in the last 24 hours?" "When did you get your first job?" "What did you do last evening?" "How are an apple and orange the same?"

"When did you get your first job?" Explanation: Asking the client about when he or she got his or her first job gives information about the client's remote memory or past events. Asking about what the client ate in the past 24 hours, or what he or she did last evening, provides information about the client's recent memory. Asking how an apple and orange are similar tests abstract reasoning

A nursing instructor is discussing mental health assessments with students. In what situations would the instructor tell the students an acute mental health assessment is necessary? A situation that involves danger of harm to self or others A situation in which the admitted client is diagnosed with schizophrenia Any time a client is severely depressed When a client is assessed as delirious

A situation that involves danger of harm to self or others Explanation: An acute mental health assessment includes questions about harm to self or others. Acute situations include a risk for injury that accompanies psychotic states, depression, dementia, or delirium. It is important to ask the safety questions first and leave the presenting problem last. Clients with schizophrenia do not always present with risk for harm to self or others. A client with severe depression is not necessarily at risk of harm to self or others nor is a delirious client.

A 55-year-old male client has just been diagnosed with presbycusis. In the interview with the client, the nurse should most expect the client to complain of having trouble hearing which of the following in the initial stages of this condition? A story his wife is telling him The bass speakers of his stereo system His son giving him directions to a restaurant The sound of his car engine starting

A story his wife is telling him Explanation: Presbycusis often begins with a loss of high-frequency sounds (woman's voice) followed later by the loss of low-frequency sounds. The bass speakers, his son's voice, and the engine starting would all have lower-frequency sounds than his wife's voice.

The nurse suspects a child is a victim of abuse. What observation caused the nurse to make this clinical determination? Hair needs to be combed Has a small hole in the shirt Wears shorts and tennis shoes Appears younger than stated age

Appears younger than stated age Explanation: Abused children may appear younger than the stated age due to developmental delays or malnourishment. Mussy hair, a hole in the shirt, and wearing shorts and tennis shoes are not observations that indicate that the child is a victim of abuse.

Which nursing assessment data cue is supportive of a diagnosis of impaired social interaction? (Select all that apply.) Argues with family Poor communication skills Avoid peers and others Inaccurate interpretation of surroundings Loneliness

Argues with family Poor communication skills Avoid peers and others Assessment data supporting a nursing diagnosis of impaired social interaction include poor communication skills, arguing with family and others, and avoiding peers and others. Inaccurate interpretation of surroundings is supportive of a nursing diagnosis of altered thought processes. Individuals at risk for suicide express loneliness.

Upon inspection of an elderly client's ears using an otoscope, the nurse observes a cloudy appearance to the tympanic membranes with prominent landmarks. What action should the nurse perform first? Perform hearing test to assess for conductive hearing loss Document these findings as normal changes in the older adult Ask whether the client is experiencing ear pain or pressure Assess for balance using the Romberg test

Ask whether the client is experiencing ear pain or pressure Explanation: The older client's eardrums may appear cloudy with prominent landmarks due to the atrophy of the tympanic membranes associated with normal aging. The nurse should ask the client about ear pain because prominent landmarks can also be a sign of negative pressure associated with obstructed eustachian tubes. An ear infection should be ruled out and problems with hearing and balance assessed before making a clinical judgment that this is a normal finding. All objective information should be documented by the nurse.

A nurse is assigned to care for a client who has been physically abused by her husband. The nurse finds that client has an abuse score of 4 in her documents. Which of the following descriptions corresponds to the abuse score? Beating up and severe contusions Punching and kicking Head injury and internal injury Threat of abuse by weapons

Beating up and severe contusions Explanation: An abuse score of 4 corresponds to beating up and severe contusions. Punching and kicking are given a score of 3. Head injury and internal injury are rated a score of 5. Threat of abuse by weapons is given a score of 1.

A nurse is inspecting the ears of an Asian client and observes that her earlobes appear soldered, or tightly attached to adjacent skin with no apparent lobe. Which of the following should the nurse do next? Notify the physician of the finding Ask the client whether she has ever experienced an injury involving her ears Continue with the examination Record the finding and plan to follow-up at the client's next visit to note any changes

Continue with the examination Explanation: Earlobes may be free, attached, or soldered (tightly attached to adjacent skin with no apparent lobe). Most African Americans and Caucasians have free lobes, whereas most Asians have attached or soldered lobes, although any type is possible in all cultural groups. Thus, this finding is normal and does not need to be reported to the physician, followed up on with a question to the client about an ear injury, or recorded and followed up on at a later visit.

An adult female client is being assessed for intimate partner violence (IPV). She acts as if the abuse she has suffered is normal. What could be a reason for her actions? She has a history of violence in her family She does not understand what constitutes violence She does not want to get into trouble She knows if she says anything she will be abused again

Correct response: She has a history of violence in her family Explanation: Some clients deny the seriousness of violence or consider it "normal." They may feel this way because many people in their family have also experienced violence or because most women or men or a specific group in their culture commonly experience it.

The nurse is caring for a woman in the prenatal clinic who comments that she just cannot seem to get things "right" anymore at home and that her husband says she knows so little about life. Which type of abusive or controlling behavior is the woman describing? Emotional abuse Minimizing Intimidation Blaming

Emotional abuse Explanation: Emotional abuse refers to the victim being put down or made to feel bad about them. The perpetrator makes light of abuse and says abuse did not occur through minimizing and blaming. Intimidation makes the victim afraid through the use of looks, action, or gestures.

The nurse would suspect a problem at which area when pressure builds up on either side of the tympanic membrane? Cochlea Eustachian tube Vestibulocochlear nerve Organ of Corti

Eustachian tube Explanation: The eustachian tube equalizes the pressure on either side of the tympanic membrane, which separates the external ear from the middle ear. The cochlea, organ of Corti, and the vestibulocochlear nerve are part of the inner ear.

When does the nurse screen for alcohol and drug use? Every client/every client history Every client/every interaction At-risk client/every client history At-risk client/initial assessment

Every client/every client history Explanation: Screening for alcohol and drug use is part of every client history.

The nurse observes a client in the day room laughing uncontrollably and within 5 minutes sitting in the corner sobbing. This behavior continues throughout the day. What term would the nurse use in documenting this behavior? Lability Elation Blunted affect Euphoria

Explanation: Lability is a quick change of expression of mood or feelings. Elation is a high degree of confidence, boastfulness, uncritical optimism, and joy accompanied by increased motor activity. Blunted affect is a severe reduction in emotional expressiveness. Euphoria is an excessive sense of emotional and physical well-being inappropriate to the actual situation or environmental stimuli.

The nurse is concerned that an older community member is being abused by an adult son however will not report the abuse or seek help. What could be a reason for this older adult to refrain from reporting the abuse? The abuse is not that bad The older adult loves the son Fear of being institutionalized The son provides companionship

Fear of being institutionalized Explanation: Elder abuse is often difficult to assess because of the older person's hesitancy to report the abuse. Reasons why abuse is not reported include the fear of being institutionalized. Other reasons why the abuse is not reported include mistrust of law enforcement, family relationship that may create feelings of guilt, burden, dependence, of fear of abandonment. It is unlikely that the abuse is not being reported because the abuse is not that bad, the older adult loves the son, or because the son provides companionship.

When inspecting the tympanic membrane, which of the following structures does the nurse expect to identify? Cone of light, incus, umbo, cochlea Pars tensa, umbo, handle of malleus, ossicles Pars tensa, pars flaccida, vestibule, cone of light Handle of malleus, short process of malleus, cone of light

Handle of malleus, short process of malleus, cone of light Explanation: Visualization of the tympanic membrane using an otoscope includes inspection of the cone of light, the short process of the malleus, and the handle of the malleus. The cochlea, vestibule, and stapes (part of the ossicles) are not normally visualizable.

The nurse is assisting a female client who is being physically abused about a safety plan. The client prefers to return home. Which of the following would the nurse need to do first? Have the client complete a danger assessment Notify the neighbors about the abuse Tell her to have her bags packed Give the client the number of a shelter

Have the client complete a danger assessment Explanation: If a client says that she prefers to return home, ask her if it is safe for her do so and have her complete a danger assessment tool. After doing so, the nurse would then help her devise a safety plan including having a bag packed, and giving her contact information for shelters and groups. The client would be urged to tell neighbors about the abuse and ask them to call the police if they hear a disturbance. This would not be something the nurse would do.

A client has anisocoria on examination. Pathological causes of this include which of the following? Horner's syndrome Benign anisocoria Differing light intensities for each eye Eye prosthesis

Horner's syndrome - A disrupted nerve pathway on one side from the brain to the face and eye. - aniscoria: unequal pupil sizes Explanation: Anisocoria can be associated with serious pathology. Remember to exclude benign causes before embarking on an intensive follow-up. Testing the near reaction in this case may help locate an Argyll-Robertson or tonic (Adie's) pupil.

A nurse assesses the respiration pattern on a client who arrives in the emergency department due to an overdose of narcotics. The nurse notes that the respirations are decreased in rate and depth, and have an irregular pattern. How should the nurse document this finding? Cheyne-Stokes respiration Biot's respiration Hypoventilation Bradypnea

Hypoventilation is decreased rate, decreased depth, and irregular pattern of respiration. A client with regular pattern characterized by alternating periods of deep, rapid breathing followed by periods of apnea has Cheyne-Stokes respiration. A client with irregular pattern characterized by varying depth and rate of respirations followed by periods of apnea has Biot's respiration. A client with bradypnea may have a regular respiration rate of less than 10/min.

A client presents with purulent, bloody drainage of the ear. Which of the following would the nurse assess first? Assess the tympanic membrane. Ask whether there is any pain in the ear. Inspect the external ear canal. Ask whether the client has experienced any hearing loss.

Inspect the external ear canal. Explanation: Purulent, bloody drainage suggests external otitis, an infection of the external ear canal. Therefore the nurse would need to inspect the external auditory canal. Assessing the tympanic membrane would be appropriate if the client has purulent drainage, pain, and complained of a popping sensation, which is associated with otitis media and tympanic perforation. Asking about pain or hearing loss are general questions that would be asked regardless of the situation.

A nurse is assessing the nutritional status of an elderly client. Which of the following tools would be best for the nurse to use for this client? 24-hour food recall Mini nutritional assessment (MNA-SF) Sample form for a nutrition history Checklist for nutritional screening

Mini nutritional assessment (MNA-SF) Explanation: For the older adult client, the Mini Nutritional Assessment (MNA-SF) is a valid stand-alone assessment tool that can be used to screen the nutritional status of older adults in less than five minutes. Using a 24-hour food recall is an efficient and easy method of identifying a client intake. However, this tool but can only be used with a person who is able to remember all types and quantities of foods and beverages taken in a 24-hour period. Additional tools used to screen a person food habits and nutrition include a sample form for a nutrition history and a checklist to use for nutritional screening.

Which characteristic of a cough should alert the nurse to assess the client for other findings of sinusitis? Onset upon awakening Nighttime when lying down Intermittent but worse in evening Persistent all day

Nighttime when lying down Explanation: A cough that occurs when lying down at night is due to sinusitis or postnasal drip. Continuous coughs are usually associated with acute infections; whereas those occurring only early in the morning are often associated with chronic bronchial inflammation or smoking. Coughs late in the evening may be the result of exposure to an irritant during the day.

The nurse's assessment of an older adult client's ears and hearing suggests the possible presence of conductive hearing loss. What is the most likely etiology of this abnormal assessment finding? Otitis media Cranial nerve VIII damage Trauma to the temporal lobe Age-related hearing changes

Otitis media Explanation: Otitis media is a contributor to conductive hearing loss. However, it does not affect the inner ear and the pathways of sensorineural hearing. Conductive hearing loss would be related to a dysfunction of the external or middle ear (e.g., impacted ear wax, otitis media, foreign object, perforated eardrum, drainage in the middle ear, or otosclerosis). A sensorineural loss would be related to dysfunction of the inner ear (i.e., organ of Corti, cranial nerve VIII, or temporal lobe of brain). Presbycusis affects sensorineural hearing more than conductive hearing ability.

A client has been diagnosed with conductive hearing loss. The nurse understands that which of the following could be the cause of this type of hearing loss? Perforated eardrum Damage to cranial nerve VIII Dysfunction of the temporal lobe of the brain Injury to the organ of Corti

Perforated eardrum Explanation: The transmission of sound waves through the external and middle ear is referred to as conductive hearing and the transmission of sound waves in the inner ear is referred to as perceptive or sensorineural hearing. Therefore, a conductive hearing loss would be related to a dysfunction of the external or middle ear (e.g., impacted ear wax, otitis media, foreign object, perforated eardrum, drainage in the middle ear, or otosclerosis). A sensorineural loss would be related to dysfunction of the inner ear (i.e., cranial nerve VIII, temporal lobe of brain, or organ of Corti).

A nurse has taught a group of older adults about the high incidence and prevalence of macular degeneration. What health promotion and prevention activity should the nurse encourage these clients to perform? Obtain a home version of the Snellen chart and test their vision weekly Rinse their eyes with a warmed, normal saline solution three to four times per week Maintain a low-sodium diet Post an Amsler grid in their home and perform the test on a regular basis

Post an Amsler grid in their home and perform the test on a regular basis Explanation: The Amsler test is used to identify the early symptoms of macular degeneration. This does not involve the use of a Snellen chart. Sodium limitation is beneficial to overall health but does not specially address the risk factors for macular degeneration. Rinsing the eyes is not necessary or relevant.

The emergency department nurse notes a clear, watery discharge from the client's ear following a bicycle accident. Which of the following actions should the nurse do next? Refer the client immediately for further evaluation. Assess for foreign body impaction. Examine for postauricular cysts. Position the client to facilitate drainage.

Refer the client immediately for further evaluation. Explanation: Watery drainage may suggest cerebrospinal fluid, for which the client should be referred immediately for further evaluation. Tophi and postauricular cysts would be visible on inspection and are not associated with drainage. Repositioning the client is not a priority, due to the potential severity of the client's injury.

What intervention will the nurse implement initially for a client who has reported experiencing unexplained, severe neck pain for more than 2 months? Request a consult with physical therapy. Provide education regarding exercises that focus on strengthening neck muscles. Screen for possible depression. Inquire about possible pain medication abuse.

Screen for possible depression. Explanation: Unexplained conditions lasting more than 6 weeks are increasingly recognized as chronic disorders that should prompt screening for depression, anxiety, or both. While the remaining options may prove to be appropriate, the screening is the initial intervention.

A client is admitted to the health care facility with a diagnosis of left lower lobe pneumonia. What change in egophony should the nurse expect to find in the left lower lobe? "Ninety nine" is soft and muffled Letter "E" is heard distinctly "1-2-3" is heard clearly Sound is louder and sounds like "A"

Sound is louder and sounds like "A" Explanation: To perform egophony, the nurse asks the client to repeat the letter "E" while listening with the stethoscope. Over normal lung tissue, the sound will be soft and muffled but the letter should be distinguishable. In areas of consolidation, such as pneumonia, the letter "E" will sound louder and sound like the letter "A". Bronchophony uses the words "Ninety nine". Whispered pectoriloquy uses the phrase "1-2-3".

A 52-year-old client fails the Romberg test. The nurse explains that this might indicate a dysfunction in what part of the ear? The bones of the middle ear The eustachian tubes The pinna The vestibular portion of the inner ear

The vestibular portion of the inner ear Explanation: Failure of the Romberg test may indicate dysfunction in the vestibular portion of the inner ear, semicircular canals, and vestibule.

A nurse is interviewing a child who is suspected of being abused. What nursing action would be most appropriate? Ask questions that are highly detailed Use direct, nonleading questions Offer the child a reward for answering questions Use simple yes and no questions regardless of the child's age

Use direct, nonleading questions Explanation: When interviewing children, the nurse should use questions that are direct to extract information without being leading. The less information supplied in the questions and the more information provided by the child increases the credibility of the information gathered. Questions need to be formulated based on the child's understanding and developmental stage. Children should not be coerced to answer questions by being offered a reward to do so.

Which of the following manifestations is the most problematic for the client diagnosed with Meniere's disease? Tinnitus Diaphoresis Vertigo Hearing loss

Vertigo Explanation: Vertigo is usually the most troublesome complaint related to Meniere's disease. Other clinical manifestations include tinnitus, diaphoresis, and hearing loss.

A nurse begins the mental status exam of an older adult. Before assessing the client's thought processes and perceptions, the nurse should first obtain the results of what other assessments? Vision and hearing Speech and facial expressions Vital signs and nutritional status Ability to follow commands and move extremities

Vision and hearing Explanation: When assessing the mental status of an older adult, the nurse should first check vision and hearing before assuming the client has a mental problem. Speech may be affected by a decrease in hearing. Vital signs and nutritional status give the nurse an impression of overall hemodynamic stability. Ability to follow commands and moving the extremities is a part of a client's cognitive ability.

A popular pain assessment scale for children is: Visual Analog Scale. Descriptive Pain Intensity Scale. FLACC Pain Assessment Scale. Memorial Pain Assessment Card.

Visual Analog Scale. Explanation: The visual analog scale is appropriate to assess pain in children.

Vitamins requiring a fat source to be absorbed and stored by the body include all of the following except: Vitamin A Vitamin B Vitamin D Vitamin E

Vitamin B Explanation: Vitamins A, D, E, and K are all fat-soluble, while vitamin B is not.

The nurse suspects that a female client is the victim of abuse. What physical assessment finding caused the nurse to come to this conclusion? Select all that apply. Welts across the back Bruises around the wrists Bruising around the left eye and neck Tan lines around the chest and lower abdomen Healing circular wounds along the inner thighs

Welts across the back Bruises around the wrists Bruising around the left eye and neck Healing circular wounds along the inner thighs Evidence of physical abuse includes welts on the back, bruises around the wrists, bruising around the eye and neck, and healing wounds along the inner thighs. Tan lines are not considered signs of physical abuse.

The nurse notes that an older client speaks rapidly and uses words that make no sense or communicate any clear meaning. When documenting this finding, the nurse should use which term to describe this client's speech? Dysphonia Dysarthria Wernicke's aphasia Cerebellar dysarthria

Wernicke's aphasia Explanation: Wernicke's aphasia is rapid speech that lacks meaning. It is caused by a lesion in the posterior superior temporal lobe. Dysphonia is a voice volume disorder, caused by an issue within the larynx or impairment of cranial nerve X. Dysarthria is a defect in the muscles that control speech. Cerebellar dysarthria is irregular uncoordinated speech caused by multiple sclerosis.

Which statement is true regarding client positioning when attempting to identify intercostal spaces during a respiratory assessment? Women should be assessed while in a supine position. Always position the client for easy access to the posterior surface of the chest. A prone position allows for accurate assessment of the anterior attachment of the 11th and 12th ribs. A man's 7th intercostal space is identified best when lying in a lateral position.

Women should be assessed while in a supine position. Explanation: It is easier to identify intercostal spaces in women when they lie down, as the supine position displaces breast tissue across the chest. The 11th and 12th ribs, the "floating ribs," have no anterior attachments. The remaining options present incorrect statements.

A nurse performs a comprehensive assessment on a newly admitted client. The nurse documents the following findings: Client has been unable to eat or drink for the past few days due to nausea and vomiting, vital signs: Temperature 99.9° F (37.7° C), pulse 100, blood pressure 90/50 mm Hg, oxygen saturation 95% on room air. What could be contributing to the client's slight increase in temperature? dehydration poor nutrition low blood pressure nausea and vomiting

dehydration Explanation: The body is made up of about 60% water. Because water is cooling, the body temperature will increase if a client is dehydrated and decrease when overhydrated. Low blood pressure is a symptom of dehydration, and increased heart rate is a compensatory mechanism in response to low blood pressure, but these are not contributing factors to this client's increase temperature. Poor nutrition, as well as nausea and vomiting, contributed to the dehydration but these are not the best answers regarding the client's temperature.

During the assessment of a client's ears, the nurse identifies the nearest auditory ossicle that can be seen through the translucent membrane. The nurse will document that which landmark is visible? handle and short process of the malleus umbo cone of light pars flaccida

handle and short process of the malleus Explanation: The nurse will document that the handle and short process of the malleus is visible, because this is the nearest auditory ossicle that can be seen through the translucent membrane. The umbo is the base of the malleus, and also serves as a center point landmark. The cone of light refers to the reflection of the otoscope light seen as a cone due to the concave nature of the membrane. The pars flaccida is the top portion of the membrane that appears to be less taut than the bottom portion.

The nurse conducts the physical examination of a client with a diagnosis of chronic obstructive pulmonary disease. The nurse observes the point of maximal impulse (PMI) is located in the epigastric region. What condition explains this finding? right ventricular hypertrophy aortic stenosis aortic insufficiency orthopnea

right ventricular hypertrophy Explanation: Hypertrophy can cause an anatomical shift of the right ventricle, leading the PMI to be palpable in the epigastric region. Aortic stenosis denotes a narrowed valvular orifice, which obstructs blood flow. Aortic insufficiency is caused by a valve that fails to fully close, allowing blood to leak backward, creating a regurgitant murmur. Orthopnea describes shortness of breath that occurs when the client is lying down. This would not be the reason the PMI would shift to be heard in the epigastric region.

When assessing the eye, the nurse recognizes which physical structure(s) of the face that can limit the normal visual field of a client? Select all that apply. the brow the medial aspect of the nose the cheek the lashes the lacrimal puncta

the brow the medial aspect of the nose the cheek The nurse recognizes that the brow, the medial aspect of the nose, and the cheek are all facial structures that normally restrict the client's visual field. The lashes and the lacrimal puncta do not normally restrict the client's field of vision.

x A nurse performs a hearing test on an elderly client. Which result should the nurse recognize as an indication that presbycusis is present? An inability to hear: whispered sounds the calling bell the tuning fork the sound of "th"

whispered sounds Explanation: The inability of the client to hear whispered sounds indicates presbycusis, which is a gradual sensorineural hearing loss due to degeneration of the cochlea or vestibulocochlear nerve, common in older clients. The inability to hear the calling bell may indicate deafness. The inability to hear the tuning fork may indicate sensorineural or conductive hearing loss.

A client tearfully admits to the nurse that her husband beats her when he drinks alcohol excessively. How should the nurse best respond to the client's statement? Thank you for implicating your husband in this illegal behavior. "It took a great deal of courage for you to tell me that." "I will take action to make sure that this never happens again." "Now that this is out in the open, you can begin to rebuild your life."

"It took a great deal of courage for you to tell me that." Explanation: The nurse should begin by validating and commending the client's action. Speaking about the husband's illegal behavior would be premature, and the nurse cannot necessarily guarantee that abuse will never again occur. It may be presumptuous to suggest that the client needs to rebuild her life.

The nurse is admitting a client to the mental health unit with a diagnosis of attempted suicide. Which is the best question for the nurse to ask first? Do you have any thoughts of wanting to harm or kill yourself? Do you hear voices that tell you what to do? On a sense of 0 to 10, with 10 being most intense, how suicidal do you feel now? Do you have a sense of hope for the future?

Do you have any thoughts of wanting to harm or kill yourself? Explanation: The priority is for the nurse would be to conduct a suicide assessment. The best question for the nurse to ask first is Do you have any thoughts of wanting to harm or kill yourself? The risk for suicide is not assessed using 0 to 10 scale. Asking about having a sense of hope for the future would be included in a spirituality assessment. The question, "Do you hear voices that tell you what to do?" assesses for auditory hallucinations.

A client presents to the health care clinic with reports of a stiff neck for the past 3 days. What objective information can the nurse obtain during the health history using inspection? Range of motion Head position Neck tenderness Thyroid size

Head position Explanation: While collecting history, the nurse would be able to inspect the client to see in what position the head was being held. Range of motion would require the nurse to give the client commands and would be performed during the physical assessment. Neck tenderness and thyroid size would require the use of palpation, not inspection, and would also be covered in the physical assessment portion of the examination.

The cone of light is located in the inner ear. middle ear. external ear. semicircular canal.

external ear. Explanation: The distinct landmarks of the tympanic membrane include the cone of light—the reflection of the otoscope light seen as a cone due to the concave nature of the membrane.

A client comes to the clinic, reporting that he woke up this morning with a painful right eye. What would be the most appropriate response from the nurse? "It is probably just allergies. If it still hurts in the morning call me." "A painful eye happens sometimes with allergies. Do you have allergies?" "You will need to see the doctor to have your eye checked." "Did you do anything different yesterday? You may have eye strain."

"You will need to see the doctor to have your eye checked." Explanation: Pain in the eye is never normal and should always be further explored.

A client asks why cerumen is important, stating, "It just clogs up the ears anyway." How should the nurse best respond? "It helps protect the delicate ear drum from loud noise that may be damaging." "It helps to keep the ear drum soft and functioning well." "It helps to amplify the sound waves through the inner ear." "It helps create the translucent, pearly color of the ear drum."

"It helps to keep the ear drum soft and functioning well." Explanation: The cerumen helps to keep the tympanic membrane soft. It does not protect against loud noises or influence sound conduction. The color of the tympanic membrane is not attributable to the presence of cerumen.

The client asks the nurse why the nurse put the tuning fork on the bone behind the ear. Which is the best response by the nurse? "It identifies a problem with the normal pathways for sound to travel to your inner ear." "It can identify if you have an inner ear problem causing disequilibrium." "It determines hearing loss caused by degeneration of nerves in your inner ear." "It can determine if you have a problem with repeated ear infections."

"It identifies a problem with the normal pathways for sound to travel to your inner ear." Explanation: Placing the tuning fork on the mastoid bone is one part of the Rinne's test, which assesses the normal pathways for sound to travel to the inner ear. Equilibrium is assessed with the Romberg test. Multiple sources of assessment data are used to determine whether hearing loss is caused by degeneration of nerves in the inner ear or repeated ear infections.

A client recently diagnosed with Grave's diseases exhibits protruded eyeballs. Which eye care instruction should the nurse discuss with this client? "Clean the eyes from the outer to inner canthus once a day." "Wear ultraviolet blocking glasses to slow the development of this condition." "Use sympathomimetic eye drops twice daily." "Wear an eyepatch and use moisturizing eye drops."

"Wear an eyepatch and use moisturizing eye drops." Explanation: Exophthalmos, or protruding eyeballs, is commonly caused by Grave's disease. Untreated exophthalmos can impair the ability of the eye to close properly and can increased dryness. The client should have regular eye exams and can wear an eyepatch and use moisturizing eye drops for dryness. Eyes should be cleaned from the inner to outer canthus as needed. Wearing UV blocking glasses does not affect the progression of this condition, but does help with cataracts. Sympathomimetic eye drops are used to dilate pupils for eye exams. These drops are not commonly prescribed for exophthalmos.

x A 53-year-old client with a sensorineural hearing loss asks the nurse how this occurred. Which of the following responses would be most appropriate? "This is not uncommon in people your age because your eardrum is hardening." "You probably have a lot of wax buildup that is interfering with hearing sounds." "Your work in the factory exposed you to loud noises over a long time." "You have a history of fluid building up in your middle ear, which is responsible."

"Your work in the factory exposed you to loud noises over a long time." Explanation: Sensorineural hearing loss is most commonly associated with prolonged exposure to loud noises or the use of ototoxic medications. Conductive hearing loss is associated with cerumen buildup and fluid in the middle ear. Conductive hearing impairment is not uncommon in the older client due to a greater incidence of cerumen buildup or atrophy or sclerosis of the tympanic membrane.

The nurse performs a Mini-Mental Status Examination of a client with altered thought processes. Which total score would indicate cognitive impairment? 30 20 28 25

20 Explanation: The Mini-Mental Status Examination (MMSE) is a scored test. A total of 23 or lower on the MMSE indicates cognitive impairment.

A nurse is caring for a client who has been intubated and is on a mechanical ventilator. Which of the following pain assessment tools should the nurse use to assess the client's pain? Numeric Rating Scale (NRS) Visual Analog Scale (VAS) Behavioral Pain Scale (BPS) Verbal Descriptor Scale (VDS)

Behavioral Pain Scale (BPS) Explanation: The Behavioral Pain Scale (BPS) is used to assess pain in acute care/trauma clients who are unable to verbally communicate by rating behaviors like facial expression; for example, relaxed = 1, grimacing = 4. The higher the number on the BPS, the more pain the client appears to be experiencing. The Numeric Rating Scale (NRS) is a Likert-type scale, in which 0 indicates "no pain" and 10 means "worst possible pain"; it is best for older adults with no cognitive impairment, and would not be appropriate for a client who is nonverbal or unable to communicate. The Visual Analog Scale (VAS) is a 100 mm paper line with "no pain" at one end and "worst possible pain" at the other end. The client must mark on the scale their pain level, which would not be possible with a client who has been intubated. The Verbal Descriptor Scale uses a numeric rating and simple phrases to determine pain level; patients must select the phrase that best describes their pain, so this would not be the best option for this patient.

A nurse cares for a client admitted after falling off a ladder onto a concrete floor. The client is not arousable and pupils are fixed and dilated. When performing a respiratory assessment, the nurse recognizes which breathing pattern as normal for clients with brain damage? Cheyne-Stokes Biot's Retractive Kussmaul

Biot's Explanation: In people with irritation or brain damage, the respiratory pattern will be irregular and characterized by varying depth and rate followed by periods of apnea, which is known as Biot's respiration. Cheyne-Stokes is a regular pattern characterized by alternating periods of deep, rapid breathing followed by periods of apnea. Retractive is not an observable pattern of respirations. Kussmaul is seen in clients with diabetic ketoacidosis and are characterized by deep but rapid respirations similar to hyperventilation.

While auscultating a client's trachea, the nurse hears a high, harsh sound with short inspiration and long expiration. How would the nurse document this finding? Vesicular breath sounds Bronchovesicular breath sounds Adventitious breath sounds Bronchial breath sounds

Bronchial breath sounds Explanation: Harsh, high-pitched sounds that are short during inspiration and long during expiration reflect bronchial breath sounds, typically heard over the trachea. Vesicular breath sounds are low, breezy, and soft; long in inspiration and short in expiration; and normally heard over the peripheral lung fields. Bronchovesicular breath sounds are moderate-pitched sounds equal during inspiration and expiration normally, heard over the major bronchi. Adventitious breath sounds are sound added to or superimposed over normal breath sounds.

A client presents at the clinic with a chief complaint of right ear pain. The nurse notes a rash in the right ear canal. What should the nurse know is a possible cause of these symptoms? Acute ossiculo-mastitis Chronic otosclerosis Acute mastoiditis Chronic otitis media

Chronic otitis media Explanation: Unusually soft wax, debris from inflammation or rash in the ear canal, or discharge through a perforated eardrum may be secondary to acute or chronic otitis media.

The nurse requests the client to come into the dining area for lunch. The client states, "I can't come right now. I can't leave the bathroom!" The nurse observes the client has been washing his hands for 30 minutes. What is this type of behavior considered? Phobia Compulsion Obsession Confabulation

Compulsion Explanation: This client is experiencing compulsions, which are repetitive mental acts or physical behaviors that a person feels driven to perform to reduce anxiety or distress, prevent a dreaded event or situation, or respond to an obsession.

A nurse is examining a 16-year-old girl who is visibly distraught. The client has a bruise on her face and tells the nurse that her boyfriend got rough with her recently. On further questioning, the client tells the nurse that her boyfriend raped her. Which of the following is the priority nursing intervention at this point? Apply ice to the bruise on the client's face to reduce swelling Conduct a forensic interview Assess the client for signs of psychological abuse Determine whether the boyfriend was abused as a child

Conduct a forensic interview Explanation: If a nurse discovers signs of sexual abuse, including rape, on assessing a client, the nurse should conduct a forensic interview to gather data for potential legal proceedings. The other answers are of lesser priority than conducting a forensic interview.

During a general routine screening, a nurse identifies sexual assault. What is a priority nursing action? Consult a sexual assault nurse examiner (SANE). Perform a Hurt, Insult, Threaten, Scream (HITS) assessment on the client. Administer the Danger Assessment tool. Ask the client if they knew the perpetrator.

Consult a sexual assault nurse examiner (SANE). Explanation: A sexual assault nurse examiner (SANE) should be consulted if sexual assault is identified. The SANE will obtain additional information and evidence that may be used in court. The HITS tool is used to assess the frequency of abuse. The Danger Assessment tool is used to determine a client's risk of injury due to intimate partner violence. Asking if the client knew the perpetrator will be done later, but the most important action is to obtain and preserve the evidence.

A nurse performs a focused neurological assessment on a client who was involved in a motor vehicle accident. The initial Glasgow Coma Scale (GCS) at the scene was recorded as 11. On arrival at the hospital, the nurse records the client's GSC as 13. What is the best action of the nurse? Continue to monitor the client. Notify the health care provider. Alert rapid response team. Place the client on high-flow oxygen.

Continue to monitor the client. Explanation: The GCS measures level of consciousness; the scale ranges from 3 to 15; 3 being the worst with decerebrate posturing and unresponsiveness, 15 being the best with no deficits. The client has a score of 13, which does not require any interventions except to continue to monitor for deterioration. Because there has been an improvement in the client's score, the nurse does not have to notify the health care provider or alert rapid response. There is no indication that oxygen is needed. However, if the client's GCS were 8 or less, the client would have to be intubated to protect the airway.

A nurse reviews the documentation of the nurse on the previous shift and finds that the client was obtunded. The nurse anticipates that the client will respond to stimulation in what manner? Opens eyes, answers the question, and falls back to sleep Opens eyes to a loud voice and answers with confusion Awakens only to a vigorous shake or painful stimuli Does not respond even to painful stimuli

Correct response: Opens eyes to a loud voice and answers with confusion Explanation: The obtunded client opens the eyes to a loud voice and answers with confusion. If the client opens eyes, answers the question, and falls back to sleep, the client is said to be lethargic. If the client awakens to a vigorous shake or painful stimuli, he is in the stupor stage. If the client is unresponsive even to painful stimuli, the client is in a coma.

An emergency department nurse has utilized the Confusion Assessment Method (CAM) in the assessment of a 79-year-old client with a new onset of urinary incontinence. This assessment tool will allow the nurse to confirm the presence of what health problem? Delirium Vascular dementia Schizophrenia Psychosis

Delirium Explanation: The CAM assesses for delirium; it does not assess for dementia, schizophrenia, or psychosis.

The nurse prepares to assess the eyes of a client who is aphasia. Which tool should the nurse use to test distance vision? E chart Jaeger test Newspaper Snellen chart

E chart Explanation: The E chart is used if the client cannot read or has a handicap that prevents verbal communication. This chart is configured just like the Snellen chart by the characters are only Es which face in all directions. The client points to the direction in which the E is facing. The Jaeger test and a newspaper are used to test near vision. The Snellen chart is used by clients who are able to read and state the letter that appears on the different lines of the chart.

Which of the following is the most important skill a nurse needs when conducting a mental status assessment? Rapid interpretive skills Effective listening skills Thorough assessment skills Well-developed writing skills

Effective listening skills Explanation: Rapid interpretation, well-honed writing skills, and thorough assessment skills are moot without empathic and focused listening.

A woman complains of cracking fissures in the corner of her mouth (cheilosis). The nurse instructs her to consume Eggs and milk Brussel sprouts and strawberries Nuts and legumes Potatoes and milk

Eggs and milk Explanation: Vitamin B2 sources are eggs and milk and signs of deficiencies are the development of cheilosis.

The nurse is providing client teaching to the mother of a 3-month-old with otitis media. What would the nurse explain is a risk factor for otitis media? Being outside in the spring Exposure to cigarette smoke Being taken out in crowds Being an only child

Exposure to cigarette smoke Explanation: Exposure to cigarette smoke, propping bottles for babies to feed, and bottle feeding in a supine position are all environmental factors that increase risk for otitis media.

An avid swimmer presents with ear pain. Her history includes pain and drainage from the left ear. On examination, she has pain when the ear, including the tragus, is manipulated. The canal is narrowed and erythematous with some white debris in the canal. The rest of the examination is normal. What diagnosis would be most appropriate for this client? Otitis media External otitis Perforation of the tympanum Cholesteatoma

External otitis Explanation: This is a classic history and examination of a client suffering from external otitis. Otitis media would not usually include pain with movement of the external ear or drainage unless the eardrum was perforated. In this case, the examination of the eardrum is recorded as normal. Cholesteatoma is a growth behind the eardrum and would not account for these symptoms. Otitis media would classically be accompanied by a bulging, erythematous eardrum.

A nurse is conducting a mental status assessment of a 70-year-old male client who is being treated for depression. What would the nurse consider when assessing the client's facial expression and eye contact? The nurse should inform the client that his facial expression is being assessed. Reduced eye contact is an age-related physiological change. Facial expression should be disregarded if the client has a diagnosed mental illness. Eye contact is strongly influenced by cultural norms.

Eye contact is strongly influenced by cultural norms. Explanation: Eye contact and facial expressions, such as smiling, differ widely between cultures. Reduced eye contact is not an age-related physiological change. Informing the client that his facial expression is being assessed will likely confound the assessment results. Mental illness does not preclude assessment of eye contact and facial expression.

A client exhibits purulent drainage in the right external ear canal. The client complains of pain that increases when the ear is touched. Which client teaching instructions should the nurse provide? Take over the counter common cold remedies to resolve the problem. Finish the entire course of antibiotic therapy. Permanent deafness is common with this condition. Tympanostomy tubes will usually fall out on their own over time.

Finish the entire course of antibiotic therapy. Explanation: The client's symptoms are consistent with otitis externa, usually treated with antibiotics and pain medication. The inflammation can cause temporary deafness, but permanent deafness is not common. Tympanostomy tubes are placed for middle ear effusions, not otitis externa.

The nurse suspects that a client is experiencing normal age-related changes in mental functioning. What assessment finding caused the nurse to come to this conclusion? Not engaging in conversation Inability to remember the date No idea where a wallet may be located Forgot the word to describe indigestion

Forgot the word to describe indigestion Explanation: A typical age-related change in mental functioning would be occasionally forgetting a word to use. Not engaging in conversation, inability to remember the date, and having no idea where a wallet may be located are signs of Alzheimer disease.

The nurse caring for a client with Ménière's disease makes what assessments? Ability to function Social support Gross hearing Level of disability

Gross hearing Explanation: The nurse assesses gross hearing and performs the Weber and Rinne tests. It also is important to determine the extent and effect of the client's disability.

The nurse needs to assess the visual, perceptual, and constructional ability of a client. Which of the following assessments should the nurse use? Have the client draw the face of a clock Perform the SLUMS exam Ask the client to pick up a pencil with the left hand, move it to the right, and then hand it to her Ask the client today's date

Have the client draw the face of a clock Explanation: Having the client draw the face of a clock is one way to assess visual, perceptual, and constructional ability. The SLUMS exam tests cognitive function. Giving directions to the client to perform a series of tasks, such as picking up and manipulating a pencil, is an assessment of concentration. Asking the client today's date is an assessment of orientation.

A nurse performs an admission assessment and notices that a client's speech is slow and the client has difficulty answering some of the questions. How can the nurse differentiate the cause of the client's slow speech? Ask the client about his education level Give the client the history form to read silently Have the client read a few sentences out loud Assess the client's hearing in both ears

Have the client read a few sentences out loud Explanation: Speech is influenced by experience, education level, and culture. If the client is having trouble with speech, the nurse should ask the client to name objects in the room, read from printed material, or write a sentence. Asking about education level may intimidate the client and project judgment by the nurse. Giving the client a history form to read silently will not assist in assessing speech. Assessing hearing does not help with assessing the ability of the client to formulate words.

A client is concerned about developing lung cancer, as he smoked for years. He tells the nurse that he quit smoking last year but wonders if there is anything else he can do to reduce his risk for developing this disease. Which of the following should the nurse recommend? Researching his family's history of lung cancer Eating a low-fat diet Having his home checked for radon Wearing SPF 30 sunscreen while outdoors

Having his home checked for radon Explanation: Measures that can reduce risk of developing lung cancer include the following: avoiding smoking cigarettes and secondhand smoke exposure, having one's home checked for radon, and seeking medical assessment for respiratory symptoms such as prolonged cough or pain in the chest area. Family history of lung cancer is a risk factor for developing it, but researching this would not help reduce one's risk. Eating a low-fat diet and wearing sunscreen may help prevent developing other types of cancer, but not lung cancer.

A nurse assesses the distant vision acuity of a client using the Snellen chart. Which action should the nurse implement to perform the test with accuracy? Position the client 12 feet away from the Snellen chart Instruct the client to read without reading glasses Instruct the client to lean forward and read the chart Ask the client to cover one eye with the hand

Instruct the client to read without reading glasses Explanation: The nurse should instruct the client to read without reading glasses to accurately test the distant vision acuity with a Snellen chart. Reading glasses blur the vision when reading in the distance, so they can interfere with the assessment. The nurse should position the client 20 feet, not 12 feet, away from the Snellen chart. The nurse should ensure that the client does not lean forward and read because it may be an unconscious attempt to see well. The client's eye should be covered with an opaque card. Covering the eye with the hand may encourage the client to peek through the fingers.

A nurse has begun a new job at a mental health facility. The supervisor is explaining to the nurse the features included in the definition of a mental disorder, according to the DSM-5. Which of the following should the supervisor mention to the nurse? Select all that apply. Is a behavioral or psychological syndrome or pattern that occurs in an individual Reflects an underlying psychobiologic dysfunction Is an expectable response to common stressors and losses Results in clinically significant distress or disability Is primarily a result of social deviance or conflicts with society

Is a behavioral or psychological syndrome or pattern that occurs in an individual Reflects an underlying psychobiologic dysfunction Results in clinically significant distress or disability The proposed DSM-5 definition for a mental disorder contains the following features: a behavioral or psychological syndrome or pattern that occurs in an individual; that reflects an underlying psychobiologic dysfunction, the consequences of which are clinically significant distress (e.g., a painful symptom) or disability (i.e., impairment in one or more important areas of functioning). The definition does not include an expectable response to common stressors and losses (e.g., the loss of a loved one) or a culturally sanctioned response to a particular event (for example, trance states in religious rituals); and a mental disorder is not primarily a result of social deviance or conflicts with society.

You are conducting an interview with a client suffering from schizophrenia. She says to you, "bunnies are cute as a button, buttons are on my shirt, shirts can be bought in a store." What is this type of thought process known as? Magical thinking Neologisms Loose associations Ideas of reference

Loose associations Explanation: In some cases a client presents several thoughts that don't make sense in conjunction with one another. This is often seen in clients with acute exacerbations of schizophrenia and is described as loose association.

A group of students is reviewing the structures of the ear in preparation for test. The students demonstrate understanding of the material when they identify which structure as part of the middle ear? Tympanic membrane Pinna Cochlea Malleus

Malleus Explanation: The middle ear consists of the ossicles, malleus, incus, and stapes. The tympanic membrane and pinna are structures of the external ear. The cochlea is an inner ear structure.

A 55-year-old bank teller comes to the office for persistent episodes of dizziness. The first episode started suddenly and lasted 3 to 4 hours. He experienced a lot of nausea with vomiting; the episode resolved spontaneously. He has had five episodes in the past weeks. He notes some intermittent tinnitus. Physical examination reveals a normal gait. The Weber test localizes to the right side, and air conduction is equal to bone conduction in the right ear. Nystagmus is present. Based on this description, what is the most likely diagnosis? Benign positional vertigo Vestibular neuronitis Meniere's disease Acoustic neuroma

Meniere's disease Explanation: Meniere's disease is characterized by the sudden onset of vertiginous episodes that last several hours to a day or more, then spontaneously resolve; the episodes then recur. Physical examination shows sensorineural hearing loss. The client does complain of tinnitus.

The nurse is assessing the hearing of an older adult. Which type of hearing problem might the nurse expect to find in the older adult? Presbycusis Tinnitus Conductive hearing loss Sensorineural hearing loss

Presbycusis Explanation: Presbycusis results from gradual degeneration of nerves and sensory hair cells of the organ of Corti and may be related either aging or use of ototoxic drugs. Tinnitus is a perception of buzzing or ringing in the ear(s) that does not correspond to an external sound. Disruption of sound wave transmission through the external or middle ear results in conductive hearing loss. Sensorineural hearing loss results from a problem from the inner ear to the auditory cortex.

Patients have different levels of tolerance for pain. Research has proven an increased sensitivity to pain in which of the following groups? Pregnant women Preterm newborns Toddlers School-age children

Preterm newborns Explanation: Preverbal newborns and infants are at risk for undertreatment for pain because of myths that they do not experience pain. Research has proven that preterm newborns have an increased sensitivity to pain compared to older children. This is thought to be because inhibitory neurotransmitters are in insufficient levels until full-term birth.

An adolescent client who suffered a crushing leg injury due to an all terrain vehicle (ATV) rollover, complains of painful tingling in the affected leg and inability to bend the knee. The client's leg is swollen. What is the nurse's priority action? Administer the prescribed non-steroid anti-inflammatory medication. Elevate the client's leg on several pillows and apply ice. Raise the head of the bed to High Fowler's position. Notify the healthcare provider immediately.

Notify the healthcare provider immediately. Explanation: If acute pain is undertreated or completely untreated, clients are at risk for harder-to-treat neuropathic pain syndromes, such as complex regional pain syndrome (CRPS). Continued painful assault on the peripheral nerves results in neuronal plasticity and transfer of the pain stimulus to the central nervous system. These syndromes are very difficult to treat. Patients who have had surgery or a crush-type injury are at high risk for developing CRPS. You should be aware that when a client with such an injury continues to complain of high levels of pain and begins to experience a subsequent loss of function, temperature sensitivity, swelling, or other skin changes (e.g., hair loss in the affected area), the client may be developing CRPS. You should also be alert for the common terms that clients use to report neuropathic pain, such as burning, painful tingling, pins and needles, and painful numbness. The client may be developing CRPS or suffering other complications from the crush injury. The healthcare provider should be notified immediately. Elevating the leg, head of bed, or administering an NSAID are all interventions that will not help this client.

A client with a history of opioid tolerance will have an altered physiologic response to the pain stimulus. The repeated use of opioids will cause the body to become more sensitive to pain. This sensitivity is known as which of the following? Opioid allergy Opioid hypersensitivity Opioid hyperalgesia Opioid reaction

Opioid hyperalgesia Explanation: Patients with a history of opioid tolerance pose pain-management challenges for nurses. These clients have an altered physiologic response to the pain stimulus. Repeated use of opioids causes the body to become more sensitive to pain. This sensitivity is called opioid hyperalgesia. Opioid allergy, hypersensitivity, and reaction are not accepted terms for this condition.

You are a pediatric nurse caring for a child who has been brought to the clinic with otitis externa. What assessment finding is characteristic of otitis externa? Tophi on the pinna and ear lobe Dark yellow cerumen in the external auditory canal Pain on manipulation of the auricle Air bubbles visible in the middle ear

Pain on manipulation of the auricle Explanation: Tophi are deposits of uric acid crystals and are generally painless; they are a common physical assessment finding in clients diagnosed with gout. Cerumen is a normal finding during assessment of the ear canal. Its presence does not necessarily indicate that inflammation is present. Pain when the nurse pulls gently on the auricle in preparation for an otoscopic examination of the ear canal is a characteristic finding in clients with otitis externa. Air bubbles in the middle ear may be visualized with the otoscope; however, these do not indicate a problem involving the ear canal. Aural tenderness or pain is not usually associated with middle ear disorders.

The nurse utilizes the Depression Questionnaire on a client who has recently moved to a long-term care facility. The total score is 22. What would the nurse to do next? Refer for further evaluation. Evaluate benefits versus risks of a mental health label. Assess further for dementia. Document this as a normal score.

Refer for further evaluation. Explanation: A score of 22 denotes very severe depression; referral is clearly warranted. Dementia is not indicated by this score.

A 2-year-old girl was brought to the ambulatory clinic by her mother who states, "She's put a pea in her ear, and I think she did it 2 days ago because that was the last time we ate them." The nurse's otoscopic examination confirms the presence of this foreign body in the girl's middle ear. How should the nurse best respond to this assessment finding? Attempt to remove the pea using sterile forceps. Irrigate the ear canal with warm tap water to remove the pea. Instruct the mother return for care if it does not fall out in the next few days. Refer the girl to her primary care provider for prompt removal of the pea.

Refer the girl to her primary care provider for prompt removal of the pea. Explanation: Rather than attempting removal, the nurse should refer any client with the presence of foreign bodies to the health care practitioner for prompt removal due to possible swelling and infection.

A client is brought to the emergency department by the spouse. The spouse answers all the questions, but the client appears nervous. The spouse refuses to leave the client's side. What is a priority nursing action? Instruct the spouse to leave. Tell the spouse to remain quiet. Remain in the room with the client. Ask the client if they are under duress.

Remain in the room with the client. Explanation: To ensure the safety of the client, the nurse must stay in the room with the client when the spouse/suspected abuser is present. Oftentimes, the abuser will accompany the abused to the hospital to exert power/authority over the abused. Instructing the spouse to leave, telling them to remain quiet, or asking the client if they are under duress while in the presence of the abuser will most likely anger the spouse/potential abuser, which will increase risk of injury to the client once they are discharged from the hospital.

A client arrives at the emergency room reporting drooping of one side of their face since early in the morning. The client denies pain and the face is asymmetrical with a left-sided droop. No other deficiencies such as weakness of the extremities or a change in gait are observed. What action should the nurse take? Request a neurology consult to determine the cause of facial muscle weakness. Request a stat computed tomography scan to rule out a stroke. Discharge the client home because there are no other symptoms. Alert the rapid response team.

Request a neurology consult to determine the cause of facial muscle weakness. Explanation: Because the client is showing signs and symptoms of a neurological condition, the best action of the nurse would be to have a neurologist assess the client and obtain a focused history to determine the possible cause and determine treatment. The client's symptoms are not indicative of a cerebrovascular accident (stroke), so there is no need to alert the rapid response team or request a computed tomography scan. The client needs to be seen by a health care provider before they can be discharged home.

A nurse performs a Rinne test on a client who relates a history of decreased hearing in the right ear. The test demonstrates that the client has conductive hearing loss in the right ear. What is the correct documentation of this test by the nurse? Right: BC greater than AC; left AC greater than BC Right: AC greater than BC; left BC greater than AC Right: BC greater than AC; left BC= AC Right: AC greater than BC; left AC= BC

Right: BC greater than AC; left AC greater than BC Explanation: With normal hearing, air conduction should be heard longer than bone conduction. With hearing loss, bone conduction becomes longer than or equal to air conduction. The correct documentation of this finding is Right: BC greater than AC; left AC greater than BC.

After assessing a client, the nurse noted the following: he was tearful, he tried to kill himself before coming into the hospital, he had no immediate plan for another suicide attempt, he was unable to concentrate, and he reported having trouble sleeping and having little or no appetite. The nurse also noted that the client's appearance was unkempt, that he spoke in a low monotone, and that he was unable to establish and maintain eye contact. Based on this information, which nursing diagnoses would be the most appropriate? Ineffective Role Performance Risk for Infection Risk for Suicide Risk for Self-Mutilation

Risk for Suicide Explanation: His history of a recent suicide attempt in conjunction with his signs of depression such as difficulty sleeping, lack of appetite, and inability to concentrate put him at risk for suicide. The information described in the nurse's observations does not support ineffective role performance, infection, or self-mutilation.

After reviewing a client's completed danger assessment questionnaire, the nurse determines that the client is in significant danger of intimate partner violence (IPV) leading to homicide. The client says that she would prefer to return home and that she does not have a safety plan. Which of the following nursing interventions should the nurse implement at this point? Select all that apply. Encourage the client to return home to avoid raising suspicion in the abuser. Schedule a follow-up appointment. Have the client complete Assessment Tool 10-2: Assessing a Safety Plan. Provide the client with contact information for shelters and groups. Encourage the client to call with any concerns. Call the police and ask them to pick up the abuser.

Schedule a follow-up appointment. Have the client complete Assessment Tool 10-2: Assessing a Safety Plan. Provide the client with contact information for shelters and groups. Encourage the client to call with any concerns. If screening for IPV is positive and the client's answers on the danger assessment questionnaire indicate a high probability for serious violence, the nurse should ask the client if she has a safety plan and where she would like to go when she leaves the nurse's agency, and should schedule a follow-up appointment and/or refer the client as appropriate. If the client says she prefers to return home, the nurse should ask her whether it is safe for her to do so and have her complete Assessment Tool 10-2. The nurse should also provide the client with contact information for shelters and groups, and encourage her to call with any concerns. The nurse should not encourage the client to return home, as it may not be safe for her to do so. The nurse should also not call the police and ask them to pick up the abuser, as there may not be sufficient legal grounds on which to arrest and incarcerate him. In doing so, the client could be put at further risk of abuse.

Susanne is a 27-year-old woman who has had headaches, muscle aches, and fatigue for the last 2 months. The nurse has completed a thorough history, examination, and laboratory workups, the results of which are normal. What would the next action be? A referral to a neurologist A referral to a rheumatologist Telling the client nothing has been found Screening for depression

Screening for depression Explanation: Although the nurse may consider referrals to help with diagnosis and treatment of this client, screening is a time-efficient way to recognize depression. This will allow her to be treated more expediently. The nurse may tell the client that no answer is clear yet, but also that he or she will not stop investigating until the client has gotten the help she needs. Research has shown that health care providers routinely fail to screen for depression.

A nurse is aware that damage to which structure of the ear leads to sensorineural hearing loss? External auditory canal Spiral organ of Corti Auditory ossicles Tympanic membrane

Spiral organ of Corti Explanation: Damage to the spiral organ of Corti leads to sensorineural hearing loss in a client. A sensorineural hearing loss results when damage is located in the inner ear. Conduction of sound waves occurs through normal pathways, but the impaired inner ear can not make the conversion into nerve impulses. The external auditory canal is a part of the external ear; therefore, it does not contribute to sensorineural hearing. Auditory ossicles and tympanic membranes are located in the middle ear and are, therefore, not a part of the sensorineural hearing pathway.

While the nurse is interviewing a client who is a victim of abuse, the client states that she blames herself for not satisfying her husband's demands. Applying Walker's Cycle of Violence theory, the client would be in which phase? Tension-building phase Acute battering phase Honeymoon phase Reconciliation phase

Tension-building phase Explanation: During the tension-building phase, the victim often engages in self-blame for failing to satisfy the unrealistic demands of the abuser. The acute battering phase involves the abuse. The honeymoon phase is described as a period of reconciliation.

A community nurse makes a scheduled home visit for a family of five, which includes the parents and three young children, on a hot, humid summer day. Which of the following observations would make the nurse suspect abuse? The male spouse speaks occasionally. The female spouse looks at the male spouse before speaking. The children are wearing long-sleeve shirts and pants. The nurse observes an above-ground pool with dirty water.

The children are wearing long-sleeve shirts and pants. Explanation: Long-sleeve shirts and pants worn in warm weather may be an attempt to cover bruising or other injuries. A pool that needs to be cleaned is not a sign of abuse. The male spouse speaking occasionally is not indicative of abuse; usually abusers apeak predominantly and control the conversation. The female spourse making eye contact with the male spouse before speaking may be an indicator of intimate partner abuse but is not the best answer.

A nurse is writing a care plan for a client who has been found to have a progressive hearing loss. What would be an appropriate outcome for this client? The client's ear canal will be free from cerumen. The client will have normal hearing. The client will maintain adequate balance. The client will explain a plan to accommodate hearing impairment.

The client will explain a plan to accommodate hearing impairment. Explanation: An outcome related to ear problems would be that the client explains plan to accommodate hearing impairment.

The nurse assists and educates clients about the structure and function of the sensory system. The nurse is showing the client the path taken by sound waves to reach the brain. The nurse dropped the note cards with the correct sequence on the floor. The sequence of steps is scrambled and need to be placed in the correct order. Which is the correct order of the steps in the path taken by sound waves to the brain? 5. The stimuli in the organ of Corti are sent to the vestibulocochlear nerve and to the temporal lobe in the cerebral cortex, where the sounds are interpreted. 1. Sound waves enter though the ear's external auditory canal and strike the tympanic membrane. 2. The tympanic membrane vibrates at various speeds in response to various pitches of sounds. 3. The ossicles within the middle ear act as a moveable bridge to transmit these vibrations to the oval window, which amplifies the sound waves. 4. The stapes vibrates against the membrane, setting the fluid of the cochlea in motion, which in turn passes on to the hair like nerve ending in the organ of Corti.

The correct order of the steps in the path taken by sound waves to the brain is that sound waves enter through the ear's external auditory canal and strike the tympanic membrane. The tympanic membrane vibrates at various speeds in response to various pitches of sounds. The ossicles within the middle ear act as a moveable bridge to transmit these vibrations to the oval window, which amplifies the sound waves. The stapes vibrates against the membrane, setting the fluid of the cochlea in motion, which in turn passes on to the hair like nerve ending in the organ of Corti. The stimuli in the organ of Corti are sent to the vestibulocochlear nerve and then to the temporal lobe in the cerebral cortex, where the sounds are interpreted.

The nurse is screening an older adult client who is exhibiting signs of depression. Which screening tool would be best for the nurse to use during the assessment? Geriatric Depression Scale Primary Care PTSD screening tool The Patient Health Questionnaire Mini-Mental State Examination

The nurse is screening an older adult client who is exhibiting signs of depression. Which screening tool would be best for the nurse to use during the assessment? Geriatric Depression Scale Primary Care PTSD screening tool The Patient Health Questionnaire Mini-Mental State Examination

The nurse is examining a 4-year-old girl who is being treated for a burn. When determining whether the burn may be the result of abuse, what assessment parameters should the nurse consider? Select all that apply. The presence of other scars on the child's skin The pattern or shape of the burn The location of the burn The child's explanation of how she got the burn The child's prognosis for recovering from the burn

The presence of other scars on the child's skin The pattern or shape of the burn The location of the burn The child's explanation of how she got the burn When assessing a child's burn, the nurse must consider evidence of pervious burns, the shape of the burn, the location of the burn, and the child's explanation. The recovery prognosis does not help the nurse ascertain the presence or absence of abuse.

A pediatric nurse is assessing a 7-year-old boy who is suspected of being the victim of psychological abuse by his stepfather. What criterion would the nurse use to determine whether the stepfather's actions constitute abuse? The stepfather's behavior is a threat to the child's well-being. The child states, "I wish my mom wouldn't have married my stepfather." The behavior is not aimed at fostering the boy's growth and development. The child states, "I get punished if I do not behave appropriately."

The stepfather's behavior is a threat to the child's well-being. Explanation: Psychological abuse is characterized by varied actions that threaten a child's well-being. Punishment is not necessarily abusive in all cases, and a dislike for a stepparent is not always rooted in abuse. A lack of consideration for growth and development is likely inappropriate, but does not always denote psychological abuse.

What are the bordering landmarks of the anterior triangle of the neck? (Mark all that apply.) The sternomastoid The mandible The midline of the neck The omohyoid muscle The clavicle

The sternomastoid The mandible The midline of the neck The anterior triangle is bordered by the mandible above, the sternomastoid laterally, and the midline of the neck medially.

Which of the following findings from the health history of a 70-year-old woman with tinnitus is likely most significant to her diagnosis? The woman takes aspirin 4 times daily to treat her rheumatoid arthritis. The woman has experienced transient ischemic attacks in recent years. The woman takes a beta-blocker to treat hypertension. The woman was diagnosed with oral cancer 12 years prior, which responded well to treatment.

The woman takes aspirin 4 times daily to treat her rheumatoid arthritis. Explanation: Excessive use of aspirin can cause tinnitus. A history of TIAs and cancer is not likely to be related, nor is the use of antihypertensives.

Which nursing action is most appropriate when caring for the client with ear pain? Select all that apply. Use a pain scale to monitor pain intensity Use music therapy as a distraction Provide pain medication as ordered Provide nutritional meals and snacks Use universal precautions at all times

Use a pain scale to monitor pain intensity Use music therapy as a distraction Provide pain medication as ordered Appropriate nursing actions for the client with ear pain include using a pain scale to monitor pain intensity, providing pain medication as ordered, and using alternative therapies, such as music therapy as a distraction. Universal precautions and nutritional meals and snacks are appropriate for a nursing diagnosis of Risk for infection.

During the physical examination of a client, a nurse detects a thick and tender temporal artery. Which additional assessment should the nurse perform to rule out the possibility of temporal arteritis? Vision acuity Facial symmetry Lymph nodes Temporomandibular joint

Vision acuity Explanation: The nurse should assess the client for vision acuity. If the temporal artery is hard, thick, and tender with inflammation, it may be due to temporal arteritis. Temporal arteritis may lead to blindness. Inspecting for facial symmetry, palpation of the lymph nodes, the temporomandibular joint are unrelated to the involvement of the temporal arteries

During a physical assessment, a client reports he has had an inflamed sore on his arm for 2 weeks with no signs of healing. The nurse notes the client also has bleeding gums. What do these findings suggest regarding the client's nutritional status? Vitamin C deficiency Vitamin D deficiency Vitamin B12 deficiency Vitamin A deficiency

Vitamin C deficiency Explanation: Slow to heal sores and bleeding gums are indications of a vitamin C deficiency. Bone pain and bow legs are indications of a vitamin D deficiency. A beefy red tongue and paresthesias are indications of a vitamin B12 deficiency. Dry flaky skin and night blindness are indications of a vitamin A deficiency.

A nurse is teaching about hygiene and the prevention of illness. When instructing clients how to clean their ears, what action should the nurse recommend? Washing with a warm, moist washcloth Gently irrigating with normal saline Cleaning with cotton-tipped applicator Irrigating with mildly soapy water

Washing with a warm, moist washcloth Explanation: A warm, moist washcloth should be used to clean the outside of the ears, but nothing (including fluids) should be inserted into the ear canal.

When assessing an older adult about possible mistreatment, which question would be appropriate for the nurse to ask first? Have you ever been abused? Did you ever signs papers you didn't understand? What is a typical day in your life like? Are you alone often at home?

What is a typical day in your life like? Explanation: The nurse would begin to assess an older adult for possible mistreatment by asking the client to tell the nurse about a typical day in his or her life. Then the nurse would ask questions related to signing papers or being alone at home. Asking the client if he or she was ever abused would be nontherapeutic and block further communication. The client may be fearful about the ramifications of reporting the abuse if it is occurring.

A child goes to the school nurse and complains of not being able to hear their teacher. What test could the school nurse perform that would indicate hearing loss? Chiming clock test Rinne test Whisper test Weber test

Whisper test Explanation: A general estimate of hearing can be made by assessing the client's ability to hear a whispered phrase or a ticking watch, testing one ear at a time. The Rinne and Weber tests distinguish sensorineural from conductive hearing loss. There is no chiming clock test.

The nurse is assessing the lymph nodes of a client's head and neck. Which anatomical region should nurse palpate to assess the posterior cervical lymph node? along the edge of the trapezius superficial to the sternocleidomastoid deep in the angle formed by the clavicle and the sternocleidomastoid in the midline a few centimeters behind the tip of the mandible

along the edge of the trapezius Explanation: The nurse should palpate the posterior cervical lymph node along the edge of the trapezius muscle. The superficial cervical lymph node can be palpated superficial to the sternocleidomastoid. The supraclavicular lymph node can be palpated deep in the angle formed by the clavicle and the sternocleidomastoid. The submental lymph node can be palpated in the midline a few centimeters behind the tip of the mandible.

During a health history, the nurse notes that an older client answers common questions with hesitancy and gaps in the flow and rhythm of words. What should the nurse now focus the assessment on to obtain more information about this finding? mood cognition aphasia abstract thinking

aphasia Explanation: If the client's speech lacks meaning or fluency, and exhibits abnormalities such as hesitancies or gaps in the flow and rhythm of words, the nurse should test for aphasia. Assessing mood, thought processes (cognition), or abstract thinking will not help determine the cause for the client inappropriately answering common questions.

The nurse is precepting a new nurse to the mental health unit. The nurse intervenes when which of the following is observed during a focused mental health assessment in a locked unit? documenting physical appearance of the client in the chart using the SLUMS examination tool to assess cognitive function testing remote memory by asking questions related to the client's childhood assessing the client's deep tendon reflexes with a reflex hammer

assessing the client's deep tendon reflexes with a reflex hammer Explanation: Assessing deep tendon reflexes is not part of a focused mental health assessment; it is performed in a neurological assessment. Documenting the physical appearance of the client in the chart, using the SLUMS examination tool, and testing remote memory are included in a mental health assessment. Essential assessment areas for mental status include appearance, general behavior, cognitive function and memory, and thought processes.

If a nurse suspects that a client is depressed, asking the client about any suicidal thoughts: will stimulate thoughts of suicide is important, but not an early priority will stimulate clients to act on suicidal ideation is important and will not stimulate the thought of suicide

is important and will not stimulate the thought of suicide Explanation: Many clinicians avoid the topic of self-harm or suicide because they worry that broaching it will implant the idea in the client's mind. There is little risk that talking about suicide with someone who is not already thinking about it will prompt him or her to do it. Consequently, the issue should be prioritized and directly addressed with clients who are or may be depressed.

The nurse is planning to perform an ear examination on an adult client. After explaining the procedures to the client, the nurse should ask the client to remain standing. show the client the otoscope. ask the client to remove his contact lenses. observe the client's response to the explanations.

observe the client's response to the explanations. Explanation: As you prepare the client for the ear examination, carefully note how the client responds to your explanations.

Transmission of sound waves in the inner ear is known as conductive hearing. tympanic hearing. neuromotor hearing. perceptive hearing.

perceptive hearing. Explanation: The transmission of sound waves in the inner ear is referred to as "perceptive" or "sensorineural hearing."

An adult client visits the clinic and complains of tinnitus. The nurse should ask the client if she has been dizzy. hypotensive. taking antibiotics. experiencing ear drainage.

taking antibiotics. Explanation: Ringing in the ears (tinnitus) may be associated with excessive earwax buildup, high blood pressure, or certain ototoxic medications (such as streptomycin, gentamicin, kanamycin, neomycin, ethacrynic acid, furosemide, indomethacin, or aspirin), loud noises, or other causes.

An emergency room nurse receives a report on a sexually abused child. The nurse expects which of the following to be present? Select all that apply. presence of hemorrhoids irritation and swelling of genitalia appears older than stated age rarely makes eye contact and mumbles quiet and relaxed during physical assessment

presence of hemorrhoids irritation and swelling of genitalia rarely makes eye contact and mumbles Hemorrhoids, irritation, and swelling of genitalia are indicators of sexual abuse in children. Abused children may exhibit signs of low self-esteem, for example, rarely making eye contact, being anxious, and having a soft passive speech. Children who are abused may appear younger than stated age due to developmental delays and may look malnourished or unkempt. A child who has been abused would likely have extreme apprehension during a physical examination and would not be quiet or relaxed.

The nurse is assessing the speech of a client during a mental status examination. Which statement by the client should the nurse document as a clanging speech pattern? "The yard is covered in gukkers." "I love flowers, I love beer, I love January, I love loving." "Peas are good. Trees are wood. I'd leave if I could." "See that nurse; it's cold in here; my mother likes pink flowers."

"Peas are good. Trees are wood. I'd leave if I could." Explanation: The nurse should document the statement, "Peas are good. Trees are wood. I'd leave if I could," as a finding consistent with clanging speech. Speech with choice of words based on sound rather than meaning, as in rhyming and punning, is referred to as clanging. Neologisms are invented or distorted words with highly idiosyncratic meanings. Echolalia is associated with the repetition of words or phrases. A pattern of speech that lacks association is said to be tangential.

A diabetes educator is teaching a group of adults about the risks to vision that result from poorly controlled blood glucose levels. Which of the following pathophysiologic processes underlies the vision loss associated with diabetes mellitus? Diabetes contributes to increased intraocular pressure. Increased blood glucose levels cause osmotic changes in the aqueous humor. Blood vessels supplying the retina become weak and bleeding occurs. Diabetes is associated with recurrent corneal infections and consequent scarring.

Blood vessels supplying the retina become weak and bleeding occurs. Explanation: In diabetic retinopathy, the vessels that feed the retina change and weaken. Eventually, they may become blocked and cause bleeding into the eye, which blocks vision. Diabetes does not directly cause an increase in pressure in the eye, osmotic changes in the aqueous humor or corneal infection.

Which lung sound possesses the following characteristics? Expiration is longer than inspiration; the sound is louder and higher in pitch with a short silence between inspiration and expiration. Bronchovesicular Vesicular Bronchial Tracheal

Bronchial Explanation: These characteristics are consistent with bronchial breath sounds. Be alert for these because they may occur elsewhere and indicate pneumonia or other pathology. The current explanation for this phenomenon is that fluid carries the sound from the trachea very well to the chest wall. This same explanation explains 'ee' to & 'aa' changes, whispered pectoriloquy, bronchophony, and others in which high-frequency sounds, normally blocked by air-filled alveoli, could be transmitted to the chest wall.

While auscultating a client's trachea, the nurse hears a high, harsh sound with short inspiration and long expiration. The nurse would document this as which of the following? Vesicular breath sounds Bronchovesicular breath sounds Adventitious breath sounds Bronchial breath sounds

Bronchial breath sounds Explanation: Harsh high pitches sounds short during inspiration and long during expiration reflect bronchial breath sounds, typically heard over the trachea. Vesicular breath sounds are low, breezy, and soft, long in inspiration and short in expiration, normally heard over the peripheral lung fields. Bronchovesicular breath sounds are moderate-pitched sounds equal during inspiration and expiration normally, heard over the major bronchi. Adventitious breath sounds are sound added or superimposed over normal breath sounds.

A school nurse observes multiple irregular bruises at different stages of healing on an elementary school-aged child. What action should the nurse take? Arrange a meeting with the parents for the following week. Report the suspected abuse to the appropriate authorities. Ask the child if it is okay to take photographs of the bruises. Ask the child how they got the bruises.

Report the suspected abuse to the appropriate authorities. Explanation: Nurses are mandatory reporters for those who cannot take care of themselves, for example, older adults, persons who are impaired, and children. If abuse is suspected the nurse is mandated to report the suspected abuse to the appropriate authorities following the chain of command, for example, speaking with their direct supervisor, then social services will be notified. The nurse should not delay an investigation of abuse by scheduling a meeting the following week because the child might be in grave danger. The nurse does not have the authority to take photographs of the child's injuries; this will be done by the appropriate authorities. The nurse may ask the child how they obtained the injuries, but the child may not tell the truth for fear of retribution, so this is not the best action for this situation.


Conjuntos de estudio relacionados

BCH 419 Alzheimer's Disease & ALS (exam 1)

View Set

Chapter 19 Functions of Blood Vessels

View Set

Business Taxation: Unit 15: S Corporations

View Set

Chapter 2 and 3 Leadership style and Shared governance

View Set

Daily Activity versus Planned Exercise

View Set